Difference between revisions of "2017 AMC 12A Problems/Problem 21"
(Add solution 1) |
(→Solution 1) |
||
Line 31: | Line 31: | ||
At this point, no more elements can be added to <math>S</math>. To see this, let | At this point, no more elements can be added to <math>S</math>. To see this, let | ||
− | <math>a_{n}x^n + a_{n-1}x^{n-1} + ... + a_{2}x^2 + a_{1}x + a_0</math> = <math>x(a_{n}x^{n-1} + a_{n-1}x^{n-2} + ... + a_{2}x + a_{1}) + a_0</math> | + | <math>a_{n}x^n + a_{n-1}x^{n-1} + ... + a_{2}x^2 + a_{1}x + a_0</math> = <math>x(a_{n}x^{n-1} + a_{n-1}x^{n-2} + ... + a_{2}x + a_{1}) + a_0 = 0</math> |
− | + | with each <math>a_i</math> in <math>S</math>. | |
Since <math>x</math> and the parenthesized term are both integers, neither can have any factors that <math>a_0</math> does not have. However, <math>a_0</math> is in <math>S</math> and every number in <math>S</math> already has all of its factors in <math>S</math>. Therefore, <math>x</math> must be in <math>S</math> and <math>S</math> cannot be expanded. <math>\{-10,-5,-2,-1,0,1,2,5,10\}</math> has <math>9</math> elements <math>\to \boxed{\textbf{(D)}}</math> | Since <math>x</math> and the parenthesized term are both integers, neither can have any factors that <math>a_0</math> does not have. However, <math>a_0</math> is in <math>S</math> and every number in <math>S</math> already has all of its factors in <math>S</math>. Therefore, <math>x</math> must be in <math>S</math> and <math>S</math> cannot be expanded. <math>\{-10,-5,-2,-1,0,1,2,5,10\}</math> has <math>9</math> elements <math>\to \boxed{\textbf{(D)}}</math> |
Revision as of 22:08, 8 February 2017
Problem
A set is constructed as follows. To begin, . Repeatedly, as long as possible, if is an integer root of some polynomial for some , all of whose coefficients are elements of , then is put into . When no more elements can be added to , how many elements does have?
Solution 1
At first, .
has root , so now .
has root , so now .
has root , so now .
has root , so now .
has root , so now .
has root , so now .
has root , so now .
At this point, no more elements can be added to . To see this, let
=
with each in .
Since and the parenthesized term are both integers, neither can have any factors that does not have. However, is in and every number in already has all of its factors in . Therefore, must be in and cannot be expanded. has elements
See Also
2017 AMC 12A (Problems • Answer Key • Resources) | |
Preceded by Problem 20 |
Followed by Problem 22 |
1 • 2 • 3 • 4 • 5 • 6 • 7 • 8 • 9 • 10 • 11 • 12 • 13 • 14 • 15 • 16 • 17 • 18 • 19 • 20 • 21 • 22 • 23 • 24 • 25 | |
All AMC 12 Problems and Solutions |
The problems on this page are copyrighted by the Mathematical Association of America's American Mathematics Competitions.